How to prove that the convergent value of a sequence is unique? Announcing the arrival of...

I got rid of Mac OSX and replaced it with linux but now I can't change it back to OSX or windows

Why are vacuum tubes still used in amateur radios?

Does the Black Tentacles spell do damage twice at the start of turn to an already restrained creature?

Can two people see the same photon?

Weaponising the Grasp-at-a-Distance spell

Why shouldn't this prove the Prime Number Theorem?

What does 丫 mean? 丫是什么意思?

Special flights

How can I prevent/balance waiting and turtling as a response to cooldown mechanics

The test team as an enemy of development? And how can this be avoided?

What order were files/directories output in dir?

Is there hard evidence that the grant peer review system performs significantly better than random?

What are the main differences between the original Stargate SG-1 and the Final Cut edition?

Where is the Next Backup Size entry on iOS 12?

Why is the change of basis formula counter-intuitive? [See details]

Can you force honesty by using the Speak with Dead and Zone of Truth spells together?

AppleTVs create a chatty alternate WiFi network

How do living politicians protect their readily obtainable signatures from misuse?

Moving a wrapfig vertically to encroach partially on a subsection title

Why datecode is SO IMPORTANT to chip manufacturers?

Did pre-Columbian Americans know the spherical shape of the Earth?

Project Euler #1 in C++

Why is std::move not [[nodiscard]] in C++20?

What is a more techy Technical Writer job title that isn't cutesy or confusing?



How to prove that the convergent value of a sequence is unique?



Announcing the arrival of Valued Associate #679: Cesar Manara
Planned maintenance scheduled April 23, 2019 at 23:30 UTC (7:30pm US/Eastern)Showing a sequence is convergent using the $epsilon$-$N$ definitionpointwise or uniformly convergent sequence of functions?Why is the constant that upper bounds every Cauchy sequence larger than the constant that bounds the Convergent sequence?Show that $(d(x_n,y_n))_n$ is convergentProve convergent sequence to the supremumProve that the sum of a convergent and a divergent sequence is divergentMy Proof: Every convergent sequence is a Cauchy sequence.Proving that a mapping between metric spaces is convergent if it has a convergent sequence and its image is convergent.Prove $(x_n)_{nin mathbb N}$ is a convergent sequence.A Cauchy sequence ${x_n}$ with infinitely many $n$ such that $x_n = c$.












1












$begingroup$


I need a little support in the following proposition:



Let $(x_n)$ be a convergene sequence in $mathbb{R}$ and $x_n rightarrow a $ and $x_n rightarrow b $ , then $a=b$



My proof is:



Consider $|a-b| = |a-x_n + x_n-b| le |a-x_n| + |x_n-b| < frac{epsilon}{2} + frac{epsilon}{2} = epsilon$
so it follows tat $|a-b| <epsilon$ then $a = b$



is that correct?










share|cite|improve this question











$endgroup$








  • 1




    $begingroup$
    The idea is correct but the proof is incomplete.
    $endgroup$
    – mfl
    Mar 26 at 0:44












  • $begingroup$
    what is missing?
    $endgroup$
    – José Marín
    Mar 26 at 0:44








  • 2




    $begingroup$
    You haven't ever defined a symbol. You didn't make any assumptions about $epsilon$, you haven't connected it with the sequence ${x_n}$, you haven't stated that $|a - b| < epsilon$ holds for all positive $epsilon$... Essentially all of the explanation of the proof is missing here.
    $endgroup$
    – T. Bongers
    Mar 26 at 0:48










  • $begingroup$
    I see. But if I add those details it will be ok, right?
    $endgroup$
    – José Marín
    Mar 26 at 0:53
















1












$begingroup$


I need a little support in the following proposition:



Let $(x_n)$ be a convergene sequence in $mathbb{R}$ and $x_n rightarrow a $ and $x_n rightarrow b $ , then $a=b$



My proof is:



Consider $|a-b| = |a-x_n + x_n-b| le |a-x_n| + |x_n-b| < frac{epsilon}{2} + frac{epsilon}{2} = epsilon$
so it follows tat $|a-b| <epsilon$ then $a = b$



is that correct?










share|cite|improve this question











$endgroup$








  • 1




    $begingroup$
    The idea is correct but the proof is incomplete.
    $endgroup$
    – mfl
    Mar 26 at 0:44












  • $begingroup$
    what is missing?
    $endgroup$
    – José Marín
    Mar 26 at 0:44








  • 2




    $begingroup$
    You haven't ever defined a symbol. You didn't make any assumptions about $epsilon$, you haven't connected it with the sequence ${x_n}$, you haven't stated that $|a - b| < epsilon$ holds for all positive $epsilon$... Essentially all of the explanation of the proof is missing here.
    $endgroup$
    – T. Bongers
    Mar 26 at 0:48










  • $begingroup$
    I see. But if I add those details it will be ok, right?
    $endgroup$
    – José Marín
    Mar 26 at 0:53














1












1








1





$begingroup$


I need a little support in the following proposition:



Let $(x_n)$ be a convergene sequence in $mathbb{R}$ and $x_n rightarrow a $ and $x_n rightarrow b $ , then $a=b$



My proof is:



Consider $|a-b| = |a-x_n + x_n-b| le |a-x_n| + |x_n-b| < frac{epsilon}{2} + frac{epsilon}{2} = epsilon$
so it follows tat $|a-b| <epsilon$ then $a = b$



is that correct?










share|cite|improve this question











$endgroup$




I need a little support in the following proposition:



Let $(x_n)$ be a convergene sequence in $mathbb{R}$ and $x_n rightarrow a $ and $x_n rightarrow b $ , then $a=b$



My proof is:



Consider $|a-b| = |a-x_n + x_n-b| le |a-x_n| + |x_n-b| < frac{epsilon}{2} + frac{epsilon}{2} = epsilon$
so it follows tat $|a-b| <epsilon$ then $a = b$



is that correct?







real-analysis






share|cite|improve this question















share|cite|improve this question













share|cite|improve this question




share|cite|improve this question








edited Mar 26 at 0:46







José Marín

















asked Mar 26 at 0:40









José MarínJosé Marín

250211




250211








  • 1




    $begingroup$
    The idea is correct but the proof is incomplete.
    $endgroup$
    – mfl
    Mar 26 at 0:44












  • $begingroup$
    what is missing?
    $endgroup$
    – José Marín
    Mar 26 at 0:44








  • 2




    $begingroup$
    You haven't ever defined a symbol. You didn't make any assumptions about $epsilon$, you haven't connected it with the sequence ${x_n}$, you haven't stated that $|a - b| < epsilon$ holds for all positive $epsilon$... Essentially all of the explanation of the proof is missing here.
    $endgroup$
    – T. Bongers
    Mar 26 at 0:48










  • $begingroup$
    I see. But if I add those details it will be ok, right?
    $endgroup$
    – José Marín
    Mar 26 at 0:53














  • 1




    $begingroup$
    The idea is correct but the proof is incomplete.
    $endgroup$
    – mfl
    Mar 26 at 0:44












  • $begingroup$
    what is missing?
    $endgroup$
    – José Marín
    Mar 26 at 0:44








  • 2




    $begingroup$
    You haven't ever defined a symbol. You didn't make any assumptions about $epsilon$, you haven't connected it with the sequence ${x_n}$, you haven't stated that $|a - b| < epsilon$ holds for all positive $epsilon$... Essentially all of the explanation of the proof is missing here.
    $endgroup$
    – T. Bongers
    Mar 26 at 0:48










  • $begingroup$
    I see. But if I add those details it will be ok, right?
    $endgroup$
    – José Marín
    Mar 26 at 0:53








1




1




$begingroup$
The idea is correct but the proof is incomplete.
$endgroup$
– mfl
Mar 26 at 0:44






$begingroup$
The idea is correct but the proof is incomplete.
$endgroup$
– mfl
Mar 26 at 0:44














$begingroup$
what is missing?
$endgroup$
– José Marín
Mar 26 at 0:44






$begingroup$
what is missing?
$endgroup$
– José Marín
Mar 26 at 0:44






2




2




$begingroup$
You haven't ever defined a symbol. You didn't make any assumptions about $epsilon$, you haven't connected it with the sequence ${x_n}$, you haven't stated that $|a - b| < epsilon$ holds for all positive $epsilon$... Essentially all of the explanation of the proof is missing here.
$endgroup$
– T. Bongers
Mar 26 at 0:48




$begingroup$
You haven't ever defined a symbol. You didn't make any assumptions about $epsilon$, you haven't connected it with the sequence ${x_n}$, you haven't stated that $|a - b| < epsilon$ holds for all positive $epsilon$... Essentially all of the explanation of the proof is missing here.
$endgroup$
– T. Bongers
Mar 26 at 0:48












$begingroup$
I see. But if I add those details it will be ok, right?
$endgroup$
– José Marín
Mar 26 at 0:53




$begingroup$
I see. But if I add those details it will be ok, right?
$endgroup$
– José Marín
Mar 26 at 0:53










1 Answer
1






active

oldest

votes


















2












$begingroup$

The idea behind your proof is correct but it lacks a certain formality. Namely, you have not specified the value of $n$ you are using when discussing $x_n$! Below, we make the proof formal.



Let $epsilon > 0$ be given, our goal is to show that $|a-b| < epsilon$. Since $x_n to a$, there exists $N_1 geq 1$ such that
$$
|x_n- a| < frac{epsilon}{2}
$$

for all $n geq N_1$. Similarly, because $x_n to b$, there is $N_2 in mathbb{N}$ with the property that
$$|x_n - b| <frac{epsilon}{2}
$$

for all $n geq N_2$. Now, if $N := max(N_1,N_2)$ and $n geq N$, we have by the triangle inequality that
begin{align*}
|a-b| leq |a-x_n| + |x_n - b| < frac{epsilon}{2} + frac{epsilon}{2} = epsilon.
end{align*}

Hence, $|a-b| < epsilon$. Using that $epsilon >0$ was arbitrary, it follows that $a=b$.






share|cite|improve this answer









$endgroup$














    Your Answer








    StackExchange.ready(function() {
    var channelOptions = {
    tags: "".split(" "),
    id: "69"
    };
    initTagRenderer("".split(" "), "".split(" "), channelOptions);

    StackExchange.using("externalEditor", function() {
    // Have to fire editor after snippets, if snippets enabled
    if (StackExchange.settings.snippets.snippetsEnabled) {
    StackExchange.using("snippets", function() {
    createEditor();
    });
    }
    else {
    createEditor();
    }
    });

    function createEditor() {
    StackExchange.prepareEditor({
    heartbeatType: 'answer',
    autoActivateHeartbeat: false,
    convertImagesToLinks: true,
    noModals: true,
    showLowRepImageUploadWarning: true,
    reputationToPostImages: 10,
    bindNavPrevention: true,
    postfix: "",
    imageUploader: {
    brandingHtml: "Powered by u003ca class="icon-imgur-white" href="https://imgur.com/"u003eu003c/au003e",
    contentPolicyHtml: "User contributions licensed under u003ca href="https://creativecommons.org/licenses/by-sa/3.0/"u003ecc by-sa 3.0 with attribution requiredu003c/au003e u003ca href="https://stackoverflow.com/legal/content-policy"u003e(content policy)u003c/au003e",
    allowUrls: true
    },
    noCode: true, onDemand: true,
    discardSelector: ".discard-answer"
    ,immediatelyShowMarkdownHelp:true
    });


    }
    });














    draft saved

    draft discarded


















    StackExchange.ready(
    function () {
    StackExchange.openid.initPostLogin('.new-post-login', 'https%3a%2f%2fmath.stackexchange.com%2fquestions%2f3162532%2fhow-to-prove-that-the-convergent-value-of-a-sequence-is-unique%23new-answer', 'question_page');
    }
    );

    Post as a guest















    Required, but never shown

























    1 Answer
    1






    active

    oldest

    votes








    1 Answer
    1






    active

    oldest

    votes









    active

    oldest

    votes






    active

    oldest

    votes









    2












    $begingroup$

    The idea behind your proof is correct but it lacks a certain formality. Namely, you have not specified the value of $n$ you are using when discussing $x_n$! Below, we make the proof formal.



    Let $epsilon > 0$ be given, our goal is to show that $|a-b| < epsilon$. Since $x_n to a$, there exists $N_1 geq 1$ such that
    $$
    |x_n- a| < frac{epsilon}{2}
    $$

    for all $n geq N_1$. Similarly, because $x_n to b$, there is $N_2 in mathbb{N}$ with the property that
    $$|x_n - b| <frac{epsilon}{2}
    $$

    for all $n geq N_2$. Now, if $N := max(N_1,N_2)$ and $n geq N$, we have by the triangle inequality that
    begin{align*}
    |a-b| leq |a-x_n| + |x_n - b| < frac{epsilon}{2} + frac{epsilon}{2} = epsilon.
    end{align*}

    Hence, $|a-b| < epsilon$. Using that $epsilon >0$ was arbitrary, it follows that $a=b$.






    share|cite|improve this answer









    $endgroup$


















      2












      $begingroup$

      The idea behind your proof is correct but it lacks a certain formality. Namely, you have not specified the value of $n$ you are using when discussing $x_n$! Below, we make the proof formal.



      Let $epsilon > 0$ be given, our goal is to show that $|a-b| < epsilon$. Since $x_n to a$, there exists $N_1 geq 1$ such that
      $$
      |x_n- a| < frac{epsilon}{2}
      $$

      for all $n geq N_1$. Similarly, because $x_n to b$, there is $N_2 in mathbb{N}$ with the property that
      $$|x_n - b| <frac{epsilon}{2}
      $$

      for all $n geq N_2$. Now, if $N := max(N_1,N_2)$ and $n geq N$, we have by the triangle inequality that
      begin{align*}
      |a-b| leq |a-x_n| + |x_n - b| < frac{epsilon}{2} + frac{epsilon}{2} = epsilon.
      end{align*}

      Hence, $|a-b| < epsilon$. Using that $epsilon >0$ was arbitrary, it follows that $a=b$.






      share|cite|improve this answer









      $endgroup$
















        2












        2








        2





        $begingroup$

        The idea behind your proof is correct but it lacks a certain formality. Namely, you have not specified the value of $n$ you are using when discussing $x_n$! Below, we make the proof formal.



        Let $epsilon > 0$ be given, our goal is to show that $|a-b| < epsilon$. Since $x_n to a$, there exists $N_1 geq 1$ such that
        $$
        |x_n- a| < frac{epsilon}{2}
        $$

        for all $n geq N_1$. Similarly, because $x_n to b$, there is $N_2 in mathbb{N}$ with the property that
        $$|x_n - b| <frac{epsilon}{2}
        $$

        for all $n geq N_2$. Now, if $N := max(N_1,N_2)$ and $n geq N$, we have by the triangle inequality that
        begin{align*}
        |a-b| leq |a-x_n| + |x_n - b| < frac{epsilon}{2} + frac{epsilon}{2} = epsilon.
        end{align*}

        Hence, $|a-b| < epsilon$. Using that $epsilon >0$ was arbitrary, it follows that $a=b$.






        share|cite|improve this answer









        $endgroup$



        The idea behind your proof is correct but it lacks a certain formality. Namely, you have not specified the value of $n$ you are using when discussing $x_n$! Below, we make the proof formal.



        Let $epsilon > 0$ be given, our goal is to show that $|a-b| < epsilon$. Since $x_n to a$, there exists $N_1 geq 1$ such that
        $$
        |x_n- a| < frac{epsilon}{2}
        $$

        for all $n geq N_1$. Similarly, because $x_n to b$, there is $N_2 in mathbb{N}$ with the property that
        $$|x_n - b| <frac{epsilon}{2}
        $$

        for all $n geq N_2$. Now, if $N := max(N_1,N_2)$ and $n geq N$, we have by the triangle inequality that
        begin{align*}
        |a-b| leq |a-x_n| + |x_n - b| < frac{epsilon}{2} + frac{epsilon}{2} = epsilon.
        end{align*}

        Hence, $|a-b| < epsilon$. Using that $epsilon >0$ was arbitrary, it follows that $a=b$.







        share|cite|improve this answer












        share|cite|improve this answer



        share|cite|improve this answer










        answered Mar 26 at 1:16









        rolandcyprolandcyp

        2,149422




        2,149422






























            draft saved

            draft discarded




















































            Thanks for contributing an answer to Mathematics Stack Exchange!


            • Please be sure to answer the question. Provide details and share your research!

            But avoid



            • Asking for help, clarification, or responding to other answers.

            • Making statements based on opinion; back them up with references or personal experience.


            Use MathJax to format equations. MathJax reference.


            To learn more, see our tips on writing great answers.




            draft saved


            draft discarded














            StackExchange.ready(
            function () {
            StackExchange.openid.initPostLogin('.new-post-login', 'https%3a%2f%2fmath.stackexchange.com%2fquestions%2f3162532%2fhow-to-prove-that-the-convergent-value-of-a-sequence-is-unique%23new-answer', 'question_page');
            }
            );

            Post as a guest















            Required, but never shown





















































            Required, but never shown














            Required, but never shown












            Required, but never shown







            Required, but never shown

































            Required, but never shown














            Required, but never shown












            Required, but never shown







            Required, but never shown







            Popular posts from this blog

            Magento 2 - Add success message with knockout Planned maintenance scheduled April 23, 2019 at 23:30 UTC (7:30pm US/Eastern) Announcing the arrival of Valued Associate #679: Cesar Manara Unicorn Meta Zoo #1: Why another podcast?Success / Error message on ajax request$.widget is not a function when loading a homepage after add custom jQuery on custom themeHow can bind jQuery to current document in Magento 2 When template load by ajaxRedirect page using plugin in Magento 2Magento 2 - Update quantity and totals of cart page without page reload?Magento 2: Quote data not loaded on knockout checkoutMagento 2 : I need to change add to cart success message after adding product into cart through pluginMagento 2.2.5 How to add additional products to cart from new checkout step?Magento 2 Add error/success message with knockoutCan't validate Post Code on checkout page

            Fil:Tokke komm.svg

            Where did Arya get these scars? Unicorn Meta Zoo #1: Why another podcast? Announcing the arrival of Valued Associate #679: Cesar Manara Favourite questions and answers from the 1st quarter of 2019Why did Arya refuse to end it?Has the pronunciation of Arya Stark's name changed?Has Arya forgiven people?Why did Arya Stark lose her vision?Why can Arya still use the faces?Has the Narrow Sea become narrower?Does Arya Stark know how to make poisons outside of the House of Black and White?Why did Nymeria leave Arya?Why did Arya not kill the Lannister soldiers she encountered in the Riverlands?What is the current canonical age of Sansa, Bran and Arya Stark?